A06Q5i please help.....3 problems

Discussion in 'SP5' started by uktous, Jul 27, 2009.

  1. uktous

    uktous Member

    please help.....I have no idea to solve the following problems

    Problems1, 2
    When I look at the exam report, there are 3 columns which I don't understand how to work out.
    1) the 5th column - Curr
    2) the 7th column - Alt Rtn
    3) the 8th column - stock

    Could you please tell me how to work out those column?
    and what those figure represent, eg what is Alt Rtn?


    Problem3
    I also have an other question...
    why there are 2 columns of stock?
     
  2. GraemeC

    GraemeC Member

    Two different methodologies

    The currency contribution is the deviation of actual currency specific performance from benchmark average currency contribution multiplied by (the opening weights minus the benchmark weights).

    The other two columns are I believe slightly different methods of calculating the returns using average money weights rather than opening weights. The sneaky part is that the first Alt Rtn entry should be 24% not 0.24. This I believe is an alternative solution set.
     
  3. uktous

    uktous Member

    hi,

    thank you very much for your reply

    is it possible to show me, by a numerical emaple using any 1 row, about how to work out the "currency contribution"?

    also i hope you could show me how to work out the Alt Rtn stock entry (column7,8) as well, by a numerical emaple using any 1 row
     
  4. Genesiss

    Genesiss Member

    If you don't mind Please do for the whole of the US full attribution analysis i.e. all the compnents of row 1
     
  5. uktous

    uktous Member


    Thank you !
    I understand how to work out the 5th column now! And I just ignored the 7th and the 8th columns.
    I got another question.
    It is about the outperformance of the fund which is equal to 6.58% = 22.81% - 16.23%
    According to the exam report, it is equal to the stock selection added + asset allocation added + currency added = 1.71% + 4.46% + 0.0085 x 4.86% = 6.61131%

    However, 22.81% - 16.23% = 6.58%
    It is less than 6.61131%

    But if I ignore the “currency add”, hence using 1.71% + 4.46%, they give 6.57.
    This suggests that total outperformance should not take account of the current add.
    Am I wrong? Or the total outperformance should be the sum of those 3 elements and the difference is just solely due to rounding?
     
  6. GraemeC

    GraemeC Member

    What the report means is that of the 4.86% Asset Allocation profit, 0.85% came from currency effects and (therefore) 4.01% comes from country selection.

     
  7. GraemeC

    GraemeC Member

    Fund Return = (300-(250-10))/(250-0.5x10)=24.49%
    Bench Return = (120/1.75)/(100/1.9)-1=30.29%
    Curr Return = 1.9/1.75-1 = 8.57%

    Using opening weights vs benchmark weights
    Alloc = (50%-50%)x(30.29%-16.23%)
    Curr = (50%-50%)x(8.57%-1.96%)
    Stock = (24.49%-30.29%)x50%

    I think the examiners report has got printer or transcription errors in it for columns 7 & 8

    Using average actual weights versus benchmark
    I can get a total fund return of 22.20% from (611-(500+0))/(500+0.5x0). Using weights of (250+0.5x(-10))/(500+0.5x0) gives a total US profit of -3.14% made up of -2.84% stock profit and -0.30% allocation profit.

    -0.30% = (49%-50%)x30.29%
    -2.84% = (24.49%-30.29%)x49%

    Applying similar across all countries gives total allocation profit of 4.34%, total stock profit of 1.63% which totals to 5.97%.

    I hope this helps



     
  8. uktous

    uktous Member

    thank you for your answer.

    I agree with -2.84% = (24.49%-30.29%)x49% is the stock profit

    However, the allocation profit seems to be incorrect, because you are using
    (Actual weight – benchmark weight) x benchmark return
    ...but not (Actual weight – benchmark weight) x (benchmark return - total benchmark return).

    Any idea?
     
  9. GraemeC

    GraemeC Member

    I think the first point to recall is that this is an alternative solution set so the logic will be slightly different.

    Arithmetically

    49%x24.49% - 50%x30.29%=-3.14%
    (actual,actual)-(benchmark,benchmark)

    if we add and subtract 49%x30.29% and rearrange we get

    49%x(24.49%-30.29%) + (49%-50%)x30.29% = -2.84%-0.30% = -3.14%
    (stock profit) + (allocation profit) = total profit (for sector)

    Intuitively it makes sense to me because if you go 1% underweight against a benchmark returning 30% you would expect an allocation loss of .3%, then the stock loss follows as underperformance of 5.8% against benchmark times the actual weighting you have chosen to invest in that sector.

    Just to complete the values, the currency contribution in this solution set is -0.1% which comes from (49%-50%)x(30.29%-20%), the 20% being the increase in the benchmark in local currency.:)

     
  10. Genesiss

    Genesiss Member

    GraemeC, Thanks a lot for your comprehensive analysis.
    I tend to find ST5 solutions very shallow on details...they assume too much and you have to grope around the dark for a very long time before you figure out some of the answers
     

Share This Page